Difference between revisions of "2019 AMC 10B Problems/Problem 6"
Ironicninja (talk | contribs) |
m (→Solution) |
||
Line 9: | Line 9: | ||
<math>n^2 + 4n - 437</math> | <math>n^2 + 4n - 437</math> | ||
− | <math>\frac{-4\pm \sqrt{16+437\cdot4}}{2} \Rightarrow \frac{-4\pm 42}{2}\Rightarrow \frac{38}{2} \Rightarrow \boxed{ | + | <math>\frac{-4\pm \sqrt{16+437\cdot4}}{2} \Rightarrow \frac{-4\pm 42}{2}\Rightarrow \frac{38}{2} \Rightarrow 1 + 9 = \boxed{C) 10}</math> |
iron | iron |
Revision as of 12:39, 14 February 2019
There is a real such that . What is the sum of the digits of ?
Solution
iron